How to change the limits of integration The 2019 Stack Overflow Developer Survey Results Are InIntegration limits of the double integral after conversion to the polar coordinatesConvergent or Divergent using LimitsHow do I solve a double integral with an absolute value?How are limits of integration changed?Converting limits of integrationHow to set the limits for Jacobian IntegrationLimits of integration in multivariable integrals during change of variablesChange of limits of integrationWhat theorem(s) is(are) used to change between the various improper integrals.how does an integral becoming negative effect limits of integration?

Limit the amount of RAM Mathematica may access?

Manuscript was "unsubmitted" because the manuscript was deposited in Arxiv Preprints

How can I create a character who can assume the widest possible range of creature sizes?

Geography at the pixel level

Unbreakable Formation vs. Cry of the Carnarium

How to create dashed lines/arrows in Illustrator

What could be the right powersource for 15 seconds lifespan disposable giant chainsaw?

How to manage monthly salary

How long do I have to send payment?

Inversion Puzzle

Can I write a for loop that iterates over both collections and arrays?

Why can Shazam do this?

Inflated grade on resume at previous job, might former employer tell new employer?

Monty Hall variation

Any good smartcontract for "business calendar" oracles?

Does it makes sense to buy a new cycle to learn riding?

It's possible to achieve negative score?

Why is Grand Jury testimony secret?

Time travel alters history but people keep saying nothing's changed

"Riffle" two strings

The difference between dialogue marks

Is "plugging out" electronic devices an American expression?

Does light intensity oscillate really fast since it is a wave?

Carnot-Caratheodory metric



How to change the limits of integration



The 2019 Stack Overflow Developer Survey Results Are InIntegration limits of the double integral after conversion to the polar coordinatesConvergent or Divergent using LimitsHow do I solve a double integral with an absolute value?How are limits of integration changed?Converting limits of integrationHow to set the limits for Jacobian IntegrationLimits of integration in multivariable integrals during change of variablesChange of limits of integrationWhat theorem(s) is(are) used to change between the various improper integrals.how does an integral becoming negative effect limits of integration?










4












$begingroup$


I am attempting to solve the integral of the following...



$$int_0^2 piint_0^inftye^-r^2rdrTheta $$



So I do the following step...



$$=2 piint_0^inftye^-r^2rdr$$



but then the next step is to substitute $s = -r^2$ which results in...



$$=2 piint_- infty^0frac12e^sds$$



The limits of integration are reversed now and the $r$ somehow results in $1/2$.



Can someone explain why this works? Why did substituting cause the limits change and result in the integration above?










share|cite|improve this question









$endgroup$







  • 1




    $begingroup$
    $$s = -r^2 implies ds = -2r dr implies -ds/2 = rdr$$ and as $r to infty$, $s to - infty$ and as $r to 0$, $s to 0$.
    $endgroup$
    – Mattos
    55 mins ago
















4












$begingroup$


I am attempting to solve the integral of the following...



$$int_0^2 piint_0^inftye^-r^2rdrTheta $$



So I do the following step...



$$=2 piint_0^inftye^-r^2rdr$$



but then the next step is to substitute $s = -r^2$ which results in...



$$=2 piint_- infty^0frac12e^sds$$



The limits of integration are reversed now and the $r$ somehow results in $1/2$.



Can someone explain why this works? Why did substituting cause the limits change and result in the integration above?










share|cite|improve this question









$endgroup$







  • 1




    $begingroup$
    $$s = -r^2 implies ds = -2r dr implies -ds/2 = rdr$$ and as $r to infty$, $s to - infty$ and as $r to 0$, $s to 0$.
    $endgroup$
    – Mattos
    55 mins ago














4












4








4





$begingroup$


I am attempting to solve the integral of the following...



$$int_0^2 piint_0^inftye^-r^2rdrTheta $$



So I do the following step...



$$=2 piint_0^inftye^-r^2rdr$$



but then the next step is to substitute $s = -r^2$ which results in...



$$=2 piint_- infty^0frac12e^sds$$



The limits of integration are reversed now and the $r$ somehow results in $1/2$.



Can someone explain why this works? Why did substituting cause the limits change and result in the integration above?










share|cite|improve this question









$endgroup$




I am attempting to solve the integral of the following...



$$int_0^2 piint_0^inftye^-r^2rdrTheta $$



So I do the following step...



$$=2 piint_0^inftye^-r^2rdr$$



but then the next step is to substitute $s = -r^2$ which results in...



$$=2 piint_- infty^0frac12e^sds$$



The limits of integration are reversed now and the $r$ somehow results in $1/2$.



Can someone explain why this works? Why did substituting cause the limits change and result in the integration above?







calculus integration limits






share|cite|improve this question













share|cite|improve this question











share|cite|improve this question




share|cite|improve this question










asked 59 mins ago









BolboaBolboa

391516




391516







  • 1




    $begingroup$
    $$s = -r^2 implies ds = -2r dr implies -ds/2 = rdr$$ and as $r to infty$, $s to - infty$ and as $r to 0$, $s to 0$.
    $endgroup$
    – Mattos
    55 mins ago













  • 1




    $begingroup$
    $$s = -r^2 implies ds = -2r dr implies -ds/2 = rdr$$ and as $r to infty$, $s to - infty$ and as $r to 0$, $s to 0$.
    $endgroup$
    – Mattos
    55 mins ago








1




1




$begingroup$
$$s = -r^2 implies ds = -2r dr implies -ds/2 = rdr$$ and as $r to infty$, $s to - infty$ and as $r to 0$, $s to 0$.
$endgroup$
– Mattos
55 mins ago





$begingroup$
$$s = -r^2 implies ds = -2r dr implies -ds/2 = rdr$$ and as $r to infty$, $s to - infty$ and as $r to 0$, $s to 0$.
$endgroup$
– Mattos
55 mins ago











2 Answers
2






active

oldest

votes


















6












$begingroup$

$s=-r^2$ gives $ds=-2rdr$ so $dr =-frac 1 2r ds$. Also, as $r$ increases from $0$ to $infty$, $s$ decreases from $0$ to $-infty$.






share|cite|improve this answer









$endgroup$








  • 1




    $begingroup$
    It should be noted that the minus sign from the substitution is then used to reverse the order of the limits.
    $endgroup$
    – John Doe
    54 mins ago






  • 1




    $begingroup$
    @JohnDoe Right. I didn't mention it explicitly but that is what I meant.
    $endgroup$
    – Kavi Rama Murthy
    50 mins ago


















0












$begingroup$

Do you really need substitution. We already know the antiderivative of $re^-r^2$ and it is $-e^-r^2over 2$






share|cite|improve this answer











$endgroup$













    Your Answer





    StackExchange.ifUsing("editor", function ()
    return StackExchange.using("mathjaxEditing", function ()
    StackExchange.MarkdownEditor.creationCallbacks.add(function (editor, postfix)
    StackExchange.mathjaxEditing.prepareWmdForMathJax(editor, postfix, [["$", "$"], ["\\(","\\)"]]);
    );
    );
    , "mathjax-editing");

    StackExchange.ready(function()
    var channelOptions =
    tags: "".split(" "),
    id: "69"
    ;
    initTagRenderer("".split(" "), "".split(" "), channelOptions);

    StackExchange.using("externalEditor", function()
    // Have to fire editor after snippets, if snippets enabled
    if (StackExchange.settings.snippets.snippetsEnabled)
    StackExchange.using("snippets", function()
    createEditor();
    );

    else
    createEditor();

    );

    function createEditor()
    StackExchange.prepareEditor(
    heartbeatType: 'answer',
    autoActivateHeartbeat: false,
    convertImagesToLinks: true,
    noModals: true,
    showLowRepImageUploadWarning: true,
    reputationToPostImages: 10,
    bindNavPrevention: true,
    postfix: "",
    imageUploader:
    brandingHtml: "Powered by u003ca class="icon-imgur-white" href="https://imgur.com/"u003eu003c/au003e",
    contentPolicyHtml: "User contributions licensed under u003ca href="https://creativecommons.org/licenses/by-sa/3.0/"u003ecc by-sa 3.0 with attribution requiredu003c/au003e u003ca href="https://stackoverflow.com/legal/content-policy"u003e(content policy)u003c/au003e",
    allowUrls: true
    ,
    noCode: true, onDemand: true,
    discardSelector: ".discard-answer"
    ,immediatelyShowMarkdownHelp:true
    );



    );













    draft saved

    draft discarded


















    StackExchange.ready(
    function ()
    StackExchange.openid.initPostLogin('.new-post-login', 'https%3a%2f%2fmath.stackexchange.com%2fquestions%2f3181696%2fhow-to-change-the-limits-of-integration%23new-answer', 'question_page');

    );

    Post as a guest















    Required, but never shown

























    2 Answers
    2






    active

    oldest

    votes








    2 Answers
    2






    active

    oldest

    votes









    active

    oldest

    votes






    active

    oldest

    votes









    6












    $begingroup$

    $s=-r^2$ gives $ds=-2rdr$ so $dr =-frac 1 2r ds$. Also, as $r$ increases from $0$ to $infty$, $s$ decreases from $0$ to $-infty$.






    share|cite|improve this answer









    $endgroup$








    • 1




      $begingroup$
      It should be noted that the minus sign from the substitution is then used to reverse the order of the limits.
      $endgroup$
      – John Doe
      54 mins ago






    • 1




      $begingroup$
      @JohnDoe Right. I didn't mention it explicitly but that is what I meant.
      $endgroup$
      – Kavi Rama Murthy
      50 mins ago















    6












    $begingroup$

    $s=-r^2$ gives $ds=-2rdr$ so $dr =-frac 1 2r ds$. Also, as $r$ increases from $0$ to $infty$, $s$ decreases from $0$ to $-infty$.






    share|cite|improve this answer









    $endgroup$








    • 1




      $begingroup$
      It should be noted that the minus sign from the substitution is then used to reverse the order of the limits.
      $endgroup$
      – John Doe
      54 mins ago






    • 1




      $begingroup$
      @JohnDoe Right. I didn't mention it explicitly but that is what I meant.
      $endgroup$
      – Kavi Rama Murthy
      50 mins ago













    6












    6








    6





    $begingroup$

    $s=-r^2$ gives $ds=-2rdr$ so $dr =-frac 1 2r ds$. Also, as $r$ increases from $0$ to $infty$, $s$ decreases from $0$ to $-infty$.






    share|cite|improve this answer









    $endgroup$



    $s=-r^2$ gives $ds=-2rdr$ so $dr =-frac 1 2r ds$. Also, as $r$ increases from $0$ to $infty$, $s$ decreases from $0$ to $-infty$.







    share|cite|improve this answer












    share|cite|improve this answer



    share|cite|improve this answer










    answered 55 mins ago









    Kavi Rama MurthyKavi Rama Murthy

    73.6k53170




    73.6k53170







    • 1




      $begingroup$
      It should be noted that the minus sign from the substitution is then used to reverse the order of the limits.
      $endgroup$
      – John Doe
      54 mins ago






    • 1




      $begingroup$
      @JohnDoe Right. I didn't mention it explicitly but that is what I meant.
      $endgroup$
      – Kavi Rama Murthy
      50 mins ago












    • 1




      $begingroup$
      It should be noted that the minus sign from the substitution is then used to reverse the order of the limits.
      $endgroup$
      – John Doe
      54 mins ago






    • 1




      $begingroup$
      @JohnDoe Right. I didn't mention it explicitly but that is what I meant.
      $endgroup$
      – Kavi Rama Murthy
      50 mins ago







    1




    1




    $begingroup$
    It should be noted that the minus sign from the substitution is then used to reverse the order of the limits.
    $endgroup$
    – John Doe
    54 mins ago




    $begingroup$
    It should be noted that the minus sign from the substitution is then used to reverse the order of the limits.
    $endgroup$
    – John Doe
    54 mins ago




    1




    1




    $begingroup$
    @JohnDoe Right. I didn't mention it explicitly but that is what I meant.
    $endgroup$
    – Kavi Rama Murthy
    50 mins ago




    $begingroup$
    @JohnDoe Right. I didn't mention it explicitly but that is what I meant.
    $endgroup$
    – Kavi Rama Murthy
    50 mins ago











    0












    $begingroup$

    Do you really need substitution. We already know the antiderivative of $re^-r^2$ and it is $-e^-r^2over 2$






    share|cite|improve this answer











    $endgroup$

















      0












      $begingroup$

      Do you really need substitution. We already know the antiderivative of $re^-r^2$ and it is $-e^-r^2over 2$






      share|cite|improve this answer











      $endgroup$















        0












        0








        0





        $begingroup$

        Do you really need substitution. We already know the antiderivative of $re^-r^2$ and it is $-e^-r^2over 2$






        share|cite|improve this answer











        $endgroup$



        Do you really need substitution. We already know the antiderivative of $re^-r^2$ and it is $-e^-r^2over 2$







        share|cite|improve this answer














        share|cite|improve this answer



        share|cite|improve this answer








        edited 52 mins ago

























        answered 54 mins ago









        HAMIDINE SOUMAREHAMIDINE SOUMARE

        1,836212




        1,836212



























            draft saved

            draft discarded
















































            Thanks for contributing an answer to Mathematics Stack Exchange!


            • Please be sure to answer the question. Provide details and share your research!

            But avoid


            • Asking for help, clarification, or responding to other answers.

            • Making statements based on opinion; back them up with references or personal experience.

            Use MathJax to format equations. MathJax reference.


            To learn more, see our tips on writing great answers.




            draft saved


            draft discarded














            StackExchange.ready(
            function ()
            StackExchange.openid.initPostLogin('.new-post-login', 'https%3a%2f%2fmath.stackexchange.com%2fquestions%2f3181696%2fhow-to-change-the-limits-of-integration%23new-answer', 'question_page');

            );

            Post as a guest















            Required, but never shown





















































            Required, but never shown














            Required, but never shown












            Required, but never shown







            Required, but never shown

































            Required, but never shown














            Required, but never shown












            Required, but never shown







            Required, but never shown







            Popular posts from this blog

            Isabella Eugénie Boyer Biographie | Références | Menu de navigationmodifiermodifier le codeComparator to Compute the Relative Value of a U.S. Dollar Amount – 1774 to Present.

            Lioubotyn Sommaire Géographie | Histoire | Population | Notes et références | Liens externes | Menu de navigationlubotin.kharkov.uamodifier« Recensements et estimations de la population depuis 1897 »« Office des statistiques d'Ukraine : population au 1er janvier 2010, 2011 et 2012 »« Office des statistiques d'Ukraine : population au 1er janvier 2011, 2012 et 2013 »Informations officiellesCartes topographiquesCarte routièrem

            Mpande kaSenzangakhona Biographie | Références | Menu de navigationmodifierMpande kaSenzangakhonavoir la liste des auteursm